You are on page 1of 21

Problem Set

Problem Set #2
Math 5322, Fall 2001
December 3, 2001
ANSWERS
i
Problem 1. [Problem 18, page 32]
Let / T(X) be an algebra, /

the collection of countable unions of sets


in /, and /

the collection of countable intersections of sets in /

. Let
0
be
a premeasure on / and

the induced outer measure.


a. For any E X and > 0 the exists A /

with E A and

(A)

(E) +.
Answer:
By denition,

(E) = inf
_

i=1

0
(A
i
) [ A
i

i=1
/, E

_
i=1
A
i
_
.
Thus, given > 0, we can nd some sequence A
i

i=1
/ so that
E

_
i=1
A
i
and

i=1

0
(A
i
) <

(E) +.
Dene a set A E by
A =

_
i=1
A
i
.
Since A is a countable union of sets in /, A /

. Since

is subadditive,

(A)

i=1

(A
i
) =

i=1

0
(A
i
) <

(E) +,
since

=
0
on /. This completes the proof.
b. If

(E) < then E is

-measurable i there exists B /

with E B
and

(B E) = 0.
Answer:
Since the -algebra /of

-measurable sets contains / and is closed under


countable unions and intersections, /

/.
For the rst part of the proof, assume that E / and

(E) < . For


each n N, we can apply the previous part of the problem to nd a set
A
n
/

such that E A
n
and

(A
n
) <

(E) + 1/n. Dene B by


B =

n=1
A
n
.
1
Then E B and B /

, since it is a countable intersection of sets in


/

. For every n, we have

(E)

(B)

(A
n
) <

(E) + 1/n.
Since n is arbitrary, we must have

(B) =

(E).
Since E B, B is the disjoint union of E and B E. These three sets are
in /, so we can use the additivity of

on / to get

(B) =

(E) +

(B E).
All three terms are nite and

(E) =

(B), so simple algebra shows

(B E) = 0.
For the second part of the proof, suppose that E X and there is a set
B /

so that E B and

(B E) = 0. As remarked above, /

/
so B is measurable. We showed in class that if F X and

(F) = 0, then
F is

-measurable. Thus, in the present situation, B E is measurable.


But E = B(BE) (check!). Since /is closed under taking set dierences,
we conclude that E is measurable. This completes the proof.
c. If
0
is -nite, the restriction

(E) < in (b) is superuous.


Answer:
First suppose that E is

-measurable and

(E) = . Since
0
is -nite
we can nd disjoint sets F
j
/, j N with
0
(F
j
) < and X =

j=1
F
j
.
Set E
j
= EF
j
. Then the sets E
j
are disjoint,

(E
j
)

(F
j
) =
0
(F
j
) <
and E =

j
E
j
.
As a rst try, we might invoke the previous part of the problem to nd sets
B
j
E
j
with B
j
/

and

(B
j
E
j
) = 0. The next step would be to
try to dene our desired set B as

j
B
j
. Unfortunately, this wont work.
Since /

is dened to be the collection of countable intersections of sets


in /

, theres no obvious reason why /

should be closed under taking


countable unions. Thus, we dont know that

j
B
j
will be in /

.
Note, however that /

is closed under countable unions. If A


i

i=1
/

,
then for each i there is a sequence
_
A
k
i
_

k=1
/ such that
A
i
=

_
k=1
A
k
i
.
But then if A =

i
A
i
, we have
A =

_
i=1
_

_
k=1
A
k
i
_
=
_
(i,k)NN
A
k
i
,
a countable union of elements of / (check!). Thus, A /

.
2
So, we proceed as follows. For each n N, we invoke part (a) to get a set
A
n
j
/

such that E
j
A
n
j
and

(A
n
j
) <

(E
j
) +
1
n2
j
.
Since A
n
j
and E
j
are measurable and have nite measure, this gives us

(A
n
j
E
j
) <
1
n2
j
.
We then dene
A
n
=

_
j=1
A
n
j
.
As remarked above, A
n
/

. Next, we claim that


A
n
E

_
j=1
(A
n
j
E
j
). ()
To see this, suppose that x A
n
E. Since x A
n
, there is some j such
that x A
n
j
. Since x / E and E
j
E, x / E
j
. Thus x A
n
j
E
j
. This
completes the proof of the claim.
From (), we have

(A
n
E)

j=1

(A
n
j
E
j
) <

j=1
1
n2
j
=
1
n
.
Now dene
B =

n=1
A
n
.
Then, B /

and E B. For every n we have

(B E)

(A
n
E) <
1
n
,
and so

(B E) = 0, which completes the rst part of the proof.


The proof of the converse implication is the same as in part (b).
Problem 2. [Problem 25, page 39]
Complete the proof of Theorem 1.19.
Thus, we want to prove that the following conditions on a set E R are
equivalent
3
a. E /

b. E = V N
1
, where V is a G

set and (N
1
) = 0.
c. E = H N
2
, where H is an F

set and (N
2
) = 0.
Here is a Lebesgue-Stieltjes measure on R and /

is its domain (the

-
measurable sets, where

is the outer measure used in the construction of ).


Answer:
The proof of the implication (a) = (b) is pretty similar to what we did in
the last problem. In general, a countable union of G

sets is not a G

set. But,
we can start with Proposition 1.18.
So, suppose that E /

. Let F
j
be a sequence of disjoint measurable
sets such that R =

j
F
j
and (F
j
) < . We can take the F
j
s to be bounded
intervals, for example. Let E
j
= EF
j
, so the E
j
s are disjoint measurable sets
whose union is E and each E
j
has nite measure.
By Proposition 1.18, for each n N and j we can nd an open set U
n
j
such
that E
j
U
n
j
and
(U
n
j
) < (E
j
) +
1
n2
j
.
Since U
n
j
= E
j
(U
n
j
E
j
) (disjoint union), we have
(U
n
j
) = (E
j
) +(U
n
j
E
j
).
All the terms in this equation are nite, so we can rearrange the equation to get
(U
n
j
E
j
) = (U
n
j
) (E
j
) <
1
n2
j
.
Dene a set U
n
by U
n
=

j
U
n
j
. Then E U
n
and U
n
is open, since any
union of open sets is open. We have
U
n
E

_
j=1
(U
n
j
E
j
)
(check), and so
(U
n
E)

j=1
(U
n
j
E
j
) <

j=1
1
n2
j
=
1
n
.
Now we dene
V =

n=1
U
n
,
4
so V is a G

set and V E. For any n, we have


(V E) (U
n
E) <
1
n
,
so we must have (V E) = 0. We have
E = V (V E) = V N
1
where V is a G

set and N
1
= V E is a nullset. This completes the proof that
(a) = (b).
We next prove that (a) = (c). Well give a direct proof, which is easier
than the proof of (a) = (b). So suppose that E /

, let F
j
be a
partition of R into sets of nite measure and let E
j
= E F
j
.
Fix j for the moment. For each n N we can apply Proposition 1.18 to get
a compact set K
n
E
j
such that
(E
j
)
1
n
< (K
n
)
and so
(E
j
K
n
) <
1
n
.
Set H
j
=

n
K
n
. Since each K
n
is closed, H
j
is an F

set and H
j
E
j
. For
each n,
E
j
H
j
E
j
K
n
so
(E
j
H
j
) (E
j
K
n
) <
1
n
.
Thus, we have (E
j
H
j
) = 0, where H
j
E
j
is an F

set. Thus, we have


E
j
= H
j
N
j
where N
j
= E
j
H
j
is a nullset.
Taking the union over j, we have
E =
_

_
j=1
H
j
_

_
j=1
N
j
_
.
Since F

is closed under countable unions (by an argument similar to the last


problem), the rst set on the right of this equation is an F

set. Of course a
countable union of nullsets is a nullset, so the second set on the right of the last
equation is a nullset. This completes the proof of (a) = (c).
Next, we prove that (b) = (a). This is pretty easy. Suppose that
E = V N
1
where V is a G

set and N
1
is a nullset. Of course a nullset is
measurable. We know that /

contains the Borel sets, and hence the G

sets.
Since the -algebra /

is closed under complements and nite intersections,


we conclude that E /

.
5
The proof that (c) = (a) is similar. Suppose E = H N
2
where H is an
F

set and N
2
is a nullset. Then N
2
/

and H /

since /

contains
the Borel sets and hence the F

sets. Since /

is closed under nite unions,


we conclude that E /

.
This completes the proof.
Several people observed that once you know (a) = (c), you can use
this and de Morgans laws to prove (a) = (b) (or vice-versa). Perhaps the
simplest proof of our Proposition would be to prove (a) = (c) as above, then
use de Morgans laws to get (a) = (b), and then do the reverse implications
as above.
Problem 3. [Problem 26, page 39]
Prove Proposition 1.20 (Use Theorem 1.18.)
Thus, we want to prove that if E /

and (E) < , then for every


> 0 there is a set A that is a nite [disjoint] union of open intervals such that
(E A) <
Here is a Lebesgue-Stieltjes measure on R. The symbol denotes the
symmetric dierence of the sets, i.e., E A = (E A) (A E).
Answer:
Suppose E /

with (E) < and let > 0 be given.


By Theorem 1.18 we can nd an open set U R such that E U and
(U) < (E)+. Since U and E have nite measure (UE) = (U)(E) < .
Since U is an open subset of R, U can be written as a countable disjoint
union of open intervals. Suppose rst that the number of intervals is innite,
say
U =

_
k=1
I
k
,
where each I
k
is an open interval. We have

k=1
(I
k
) = (U) < .
Since this series converges, there is some n such that

k=n+1
(I
k
) < . ()
We dene A by
A =
n
_
k=1
I
k
.
6
If there are only nitely many intervals in U, we can label them as I
1
, I
2
, . . . , I
n
and let I
k
= for k n + 1. Then we dene A = U and () still holds.
We now have E A U A so
(E A) (U A)
= (U) (A)
=

k=1
(I
k
)
n

k=1
(I
k
)
=

k=n+1
(I
k
) < .
On the other hand, A E U E, so
(A E) (U E) < .
Thus,
(E A) (E A) +(A E) < 2.
Since > 0 was arbitrary, the proof is complete.
Problem 4. [Problem 28, page 39]
Let F be increasing and right continuous, and let
F
be the associated
measure. Then
F
( a ) = F(a) F(a),
F
([a, b)) = F(b) F(a),

F
([a, b]) = F(b) F(a) and
F
((a, b)) = F(b) F(a).
Answer:
Recall that
F
is constructed so that on the algebra of h-intervals it takes the
values

F
((a, b]) = F(b) F(a),
and (hence) that
F
is nite on bounded subsets of R.
To calculate
F
( a ), note that
a =

n=1
(a 1/n, a]. (A)
Certainly a is in the right-hand side and any number bigger that a is not. If
x < a then x < a 1/n for suciently large n, so x is not in the intersection
on the right-hand side of (A).
The intervals (a1/n, a] form a decreasing sequence of sets, so by continuity
from above (Theorem 1.8d) we have

F
( a ) = lim
n

F
((a 1/n, a]) = lim
n
[F(a) F(a 1/n)].
7
Since F is increasing,
lim
n
F(a 1/n) = lim
xa

F(x) = sup F(x) [ x < a = F(a).


Thus, we have

F
( a ) = F(a) F(a). (B)
Next, consider
F
([a, b]). We have [a, b] = a (a, b] (disjoint union), so

F
([a, b]) =
F
( a ) +
F
((a, b])
= F(a) F(a) +F(b) F(a)
= F(b) F(a)
so

F
([a, b]) = F(b) F(a) (C)
Next, consider
F
((a, b)), where we have to allow a and/or b to be innity.
If b is innity, then (a, ) is an h-interval and the denition of
F
gives

F
((a, )) = F() F(a).
The proposed formula

F
((a, b)) = F(b) F(a) (D)
is correct because F() = lim
x
F(x). If a = and b = then (a, b) =
(, ) is an h-interval and the denition of
F
gives

F
((, )) = F() F(),
which ts into formula (D) again.
If both a and b are nite, we have (a, b) = (a, b] b , so

F
((a, b)) =
F
((a, b])
F
( b )
= F(b) F(a) [F(b) F(b)]
= F(b) F(a),
so (D) holds.
Finally, consider intervals of the form [a, b), where b might be innity. We
can write [a, b) = a (a, b) (disjoint union) so

F
([a, b)) =
F
( a ) +
F
(a, b)
= F(a) F(a) +F(b) F(a)
= F(b) F(a),
Note that this is valid if F(b) = F() = .
Problem 5. [Problem 29, page 39]
Let E be a Lebesgue measurable set.
8
a. If E N, where N is the nonmeasurable set described in Section 1.1, then
m(E) = 0.
Answer:
Recall the construction in Section 1.1. We dene an equivalence relation
on [0, 1) by x y if x y Q. We let N be a set that contains exactly
one element from each equivalence class (using the Axiom of Choice). Set
R = Q [0, 1). For any set S [0, 1) and r in R we dene
S
r
= x +r [ x S [0, 1 r) x +r 1 [ x S [1 r, 1) .
We can then argue that if S is measurable, m(S
r
) = m(S), using the
translation invariance of Lebesgue measure. We also argued that
[0, 1) =
_
rR
N
r
, (disjoint union).
If N was measurable, we would have
1 = m([0, 1)) =

n=1
m(N),
which is impossible since the right-hand side can only be 0 or .
For the rst part of the present problem, we want to show that if E N
is Lebesgue measurable, then m(E) = 0. Well, for each r R we have
E
r
N
r
, as dened above. Since the sets N
r
are pairwise disjoint,
F =
_
rR
E
r
is a countable disjoint union of measurable sets contained in [0, 1). Thus,
we have
1 m(F) =

rR
m(E
r
) =

i=1
m(E).
If m(E) > 0, the sum on the right would be , so we must have m(E) = 0.
b. If m(E) > 0, then E contains a non-measurable set. (It suces to assume
that E [0, 1). In the notation of Section 1.1, E =

rR
E N
r
.)
Answer:
Briey, if E R and m(E) > 0, we can write
E =

_
n=
E [n, n + 1), (disjoint union).
9
Since m(E) > 0, at least one of the sets E [n, n + 1) must have nonzero
measure. Select one such set F = E [n, n + 1). If F contains a nonmea-
surable set, so does E. The set F n is contained in [0, 1) and has the same
measure as F. If we prove that every subset of [0, 1) of nonzero measure
contains a nonmeasurable set, then F n will contain a non-measurable
set A. But then A+n F is nonmeasurable (if A+n was measurable, so
would be (A+n) n = A).
Thus, it will suce to consider the case where E [0, 1) and m(E) > 0.
First, observe that the process weve dened above that sends S to S
r
is
invertible. Indeed, S
0
= S and for t (0, 1) we dene
t
: [0, 1) [0, 1) by

t
(x) =
_
x +t, x [0, 1 t)
x +t 1, x [1 t, 1).
Then S
t
=
t
(S). It is easy to check that
t
([0, 1 t)) = [t, 1) and
t
([1
t, 1)) = [0, t). It is then easy to check that the inverse of
t
is
1t
. Thus
(S
t
)
1t
= S.
From the rst part of the problem, we can conclude that if F is measurable
and F N
r
for some r then m(F) = 0. To see this, note that if F N
r
then F
1r
N. As weve argued before, F
1r
is measurable and m(F
1r
) =
m(F). From the rst part of the problem, m(F
1r
) = 0, so we conclude
that m(F) = 0.
Now suppose that E [0, 1) and m(E) > 0. The interval [0, 1) is the
disjoint union of the sets N
r
for r R, so E is the disjoint union of the sets
E N
r
. If all of the sets E N
r
were measurable, we would have
m(E) =

rR
m(E N
r
) =

rR
0 = 0,
which contradicts the assumption that m(E) > 0. Thus at least one of the
sets E N
r
must be nonmeasurable and we conclude that E contains a
nonmeasurable set.
Before going into the problems on Page 48. of the book, lets recall some of
the denitions involved.
Let (X, /) be a measurable space, so / T(X) is a -algebra. If A X,
we can dene a -algebra /[
A
T(A) by
/[
A
= E A [ E / . (0.1)
If A itself is in /, we have
/[
A
= E [ E A, E / .
10
If (Y, ^) is a measurable space and f : X Y , we say that f is measurable
on A X if f[
A
: A Y (the restriction of f to A) is measurable for the
-algebra /[
A
, which means that (f[
A
)
1
(N) = f
1
(N) A /[
A
for all
N ^. If A /, then f is measurable on A i f
1
(N) A / for all
N ^. Clearly, if f is a measurable function X Y , then f is measurable on
every subset of A of X since, in this case, f
1
(N) A /[
A
for all ^, because
f
1
(N) /.
The following proposition is pretty easy, but useful.
Proposition 0.1. Let (X, /) and (Y, ^) be measurable spaces and let f : X
Y be a function. Let E

A
be a countable (nite or innite) collection of
measurable subsets of X such that
X =
_
A
E

.
Then, f is measurable if and only if f is measurable on each E

, i.e., f is
measurable if and only if for each N ^, f
1
(N) E

/ for all A.
Proof. If f is measurable, then f
1
(N) /for all N ^. Since each E

/,
we have f
1
(N) E

/.
Conversely, suppose that for each N ^, f
1
(N) E

is measurable for
all . Then
f
1
(N) =
_
A
f
1
(N) E

is a countable union of sets in /, so f


1
(N) /. Thus, f is measurable.
Problem 6. [Problem 1, page 48]
Let (X, /) be a measurable space. Let f : X R and Y = f
1
(R). Then
f is measurable i f
1
( ) /, f
1
( ) / and f is measurable on
Y .
Answer:
Suppose rst that f is measurable. Then f is measurable on Y , as discussed
above. The sets and are closed sets, hence Borel sets, in R, so
f
1
( ) and f
1
( ) are measurable.
For the second part of proof suppose f
1
( ) and f
1
( ) are mea-
surable and f is measurable on Y . The sets f
1
( ), f
1
( ) and
Y form a partition of X. Since f
1
( ) and f
1
( ) are measurable,
Y = X (f
1
( ) f
1
( )) is measurable. To show f is measurable
it will suce to show it is measurable on the three sets in our partition. Of
course, f is measurable on Y by assumption. On the set f
1
( ), f is
constant and a constant function is always measurable. Similarly, f is constant,
and hence measurable, on f
1
( ). This completes the proof.
11
Problem 7. [Problem 2, page 48]
Let (X, /) be a measurable space. Suppose that f, g : X R are measur-
able.
a. fg is measurable (where 0 () = 0).
Answer:
Well consider two solutions, in slightly dierent spirits.
First Solution. Since f is measurable, we can partition X into the mea-
surable sets F
1
= f
1
(), F
2
= f
1
((, 0)), F
3
= f
1
(0), F
4
=
f
1
((0, )) and F
5
= f
1
(). (f
1
(a) is the same thing as f
1
( a ).) We
have a similar partition G
1
= g
1
(), G
2
= g
1
((, 0)), G
3
= g
1
(0),
G
4
= g
1
((0, )), G
5
= g
1
() for g. Taking pairwise intersections, we get
a partition of X into the 25 measurable sets (dont panic!) F
i
G
j
. To show
fg is measurable it will suce to prove that fg is measurable on each of the
sets F
i
G
j
.
Observe that fg is constant on the set F
1
G
1
= f
1
() g
1
().
Indeed fg is constant on all of the sets F
1
G
j
, j = 1, . . . , 5. Similarly, fg
is constant on F
5
G
j
, j = 1, . . . , 5 and F
i
G
1
, i = 1, . . . 5 and F
i
G
5
,
i = 1, . . . , 5.
This leaves the nine sets F
i
G
j
, i, j = 2, 3, 4 to consider. But the union of
these sets is S = f
1
(R) g
1
(R). Of course, f and g are real valued on S
and so fg is measurable on S by Proposition 2.6 (p. 45). This completes the
solution.
Second Solution We try to follow the proof of Proposition 2.6 on page 45
of the book. Thus, we dene F : X R R by F(x) = (f(x), g(x)). As
discussed in the book, this is measurable.
Next we dene : R R R by (x, y) = xy (with the 0 () = 0
convention). Note that is not continuous at the four points (0, ),
(, 0).
Nonetheless, we claim that is Borel measurable. To see this, dene A
R R by
A = (0, ), (0, ), (, 0), (, 0) .
This is a closed subset of RR, hence a Borel subset. Thus, B = RR A
is Borel.
Recall from our discussion in class that if Z is a metric space, and W is a
Borel subset of Z,
(B
Z
)[
W
= E W [ E B
Z
= B
W
.
Thus, the principal of the Proposition we proved above applies: If Z is the
union of two Borel sets V and W then f : Z R is Borel measurable if and
only if f is Borel measurable on V and W.
12
Thus, in order to show that is Borel measurable, it will suce to show it
is Borel measurable on A and B. On A, is constant (with value 0) and
so Borel measurable. On B, is continuous, and hence Borel measurable.
This proves the claim that is Borel measurable.
To complete the solution, let h = fg. Then h = F. Let B R be a
Borel set. Since is Borel measurable,
1
(B) is Borel in R R. Since F
is measurable, F
1
(
1
(B)) /. Thus,
h
1
(B) = ( F)
1
(B) = F
1
(
1
(B)) /,
which shows that h is measurable.
b. Fix a R and dene h(x) = a if f(x) = g(x) = and h(x) = f(x)+g(x)
otherwise. Then h is measurable.
Answer:
First Solution. We can partition X into the measurable sets
f
1
() g
1
() (A.1)
f
1
() g
1
(R) (A.2)
f
1
() g
1
() (A.3)
f
1
(R) g
1
() (A.4)
f
1
(R) g
1
(R) (A.5)
f
1
(R) g
1
() (A.6)
f
1
() g
1
() (A.7)
f
1
() g
1
(R) (A.8)
f
1
() g
1
() (A.9)
and it will suce to prove that h is measurable on each of these sets. On
the sets (A.3) and (A.7), h is constant, with value a, by our denition. On
the sets (A.1), (A.2), (A.4), (A.6), (A.8), and (A.9), h is constant (with
value either ). Finally, on the set f
1
(R) g
1
(R) in (A.5), both f
and g are real valued and h coincides with f + g, which is measurable by
Proposition 2.6. This completes the solution.
Second Solution. As in the rst part of the problem, dene F : X RR
by F(x) = (f(x), g(x)). As discussed in the book, this is measurable. Let
A R be dened by
A = (, ), (, ) ,
13
which is closed, and hence Borel, in RR. Let B = RR A, which is also
Borel. Dene : R R R by
(x, y) =
_
a, (x, y) A
x +y, (x, y) B.
This function is not continuous at the points in A, but it is nonetheless Borel
measurable. Of course, is continuous, hence Borel measurable, on B, and
is constant, hence Borel measurable, on A. Thus, is Borel measurable
on R R.
Since is Borel measurable, F is measurable (as discussed in the solution
of the rst part of the problem), but h = F, so the proof is complete.
Problem 8. [Problem 3, page 48]
Let (X, /) be a measurable space.
If f
n
is a sequence of measurable functions on X, then x [ limf
n
exists
is a measurable set.
Answer:
The problem is not stated very well, since it leaves some ambiguity about where
the values of the f
n
s are supposed to be. Going by the previous problems, Id
say we should consider functions with values in R.
Dene E = x [ limf
n
exists . By Proposition 2.7, the functions g, h: X
R dened by
g(x) = liminf
n
f
n
(x)
h(x) = limsup
n
f
n
(x)
are measurable and, of course,
E = x X [ g(x) = h(x) ,
so it will suce to prove that the set where these two measurable functions are
equal is measurable.
For a rst attempt, one could try to dene (x) = h(x) g(x), claim that
is measurable and observe that E =
1
(0). Its not quite that easy, since
there may be points where h(x) g(x) is undened, e.g., if h(x) = g(x) = .
We can take care of this in the spirit of some previous solutions. We can
14
partition X into the following measurable sets
S
1
= g
1
() h
1
() (A.1)
S
2
= g
1
() h
1
(R) (A.2)
S
3
= g
1
() h
1
() (A.3)
S
4
= g
1
(R) h
1
() (A.4)
S
5
= g
1
(R) h
1
(R) (A.5)
S
6
= g
1
(R) h
1
() (A.6)
S
7
= g
1
() H
1
() (A.7)
S
8
= g
1
() h
1
(R) (A.8)
S
9
= g
1
() h
1
() (A.9)
It will suce to show that E S
j
is measurable for j = 1, . . . , 9 (since then
E is a nite union of measurable sets).
In case (A.1), we have E S
1
= S
1
, which is measurable.
In case (A.2), E S
2
= , since g(x) ,= h(x) for all x S
2
.
In case (A.3), E S
3
= .
In case (A.4), E S
4
= .
In case (A.5), both g and h are real-valued on S
5
, so E S
5
= (h g)
1
(0),
which is measurable, since h g is measurable on S
5
by Proposition 2.6.
In case (A.6), E S
6
= .
In case (A.7), E S
7
= .
In case (A.8), E S
8
= .
Finally, in case (A.9), E S
9
= S
9
, which is measurable.
This completes the solution.
Alternate Solution. Just for fun, heres another way to do it. Since g h,
X is the disjoint union of E = x [ g(x) = h(x) and F = x [ g(x) < h(x) .
Since the complement of a measurable set is measurable, it will suce to show
that F is measurable.
We claim
F =
_
rQ
x [ g(x) < r x [ r < h(x) . ()
To see this, suppose rst that p F. Then g(p) < h(p) so there is some rational
s so that g(p) < s < h(p). But then
p x [ g(x) < s x [ s < h(x) ,
which is one of the sets in the union in ().
Conversely, if p is in the union in (), then there is some r Q so that
p x [ g(x) < r x [ r < h(x) .
15
But then g(p) < r and r < h(p), so g(p) < h(p), i.e., p F. This completes the
proof of the claim.
Of course, each of the sets
x [ g(x) < r x [ r < h(x) = g
1
([, r)) h
1
((r, ])
is measurable, so () shows that F is a countable union of mensurable sets,
hence measurable.
Problem 9. [Problem 4, page 48]
Let (X, /) be a measurable space.
If f : X R and f
1
((r, ]) / for each r Q, then f is measurable.
Answer:
As remarked in the book on page 45, the Borel algebra B
R
of R is generated
by the rays (a, ] for a R. Hence, to show f is measurable, it will suce to
show that f
1
((a, ]) is measurable for each a R.
To do this, let a R be xed but arbitrary. Since the rationals are dense in
R, we can nd a sequence of rationals r
n
that decrease to a, i.e., r
n
a and
the r
n
s form a decreasing sequence.
We claim that
(a, ] =

_
n=1
(r
n
, ]. ()
To see this, rst suppose x (a, ]. Then x > a and we can nd an open
interval U around a that does not contain x. Since r
n
a, there is some N
such that r
n
U for n N. But then r
n
< x, so x (r
n
, ], for n N. Thus,
x is in the union on the right hand side of ().
Conversely, if x is in the union in (), then x (r
n
, ] for some n. But then
a < r
n
< x, so x (a, ]. This completes the proof of the claim.
From (), we conclude that
f
1
((a, ]) =

_
n=1
f
1
((r
n
, ]).
Since r
n
is rational, f
1
((r
n
, ]) is measurable by our hypothesis. Thus,
f
1
((a, ]) is a countable union of measurable sets, and so is measurable. This
completes the proof.
Problem10. [Problem 9, page 48]
Let f : [0, 1] [0, 1] be the Cantor function and let g(x) = f(x) +x.
16
a. g is a bijection from [0, 1] to [0, 2] and h = g
1
is continuous from [0, 2] to
[0, 1].
Answer:
Since the Cantor function is continuous, g is continuous.
Recall that the Cantor function is nondecreasing, i.e., if x < y then f(x)
f(y). But then if x < y, g(x) = f(x) + x < f(y) + y = g(y). Thus, g is
strictly increasing and hence one-to-one. Since f(0) = 0 and f(1) = 1, we
have g(0) = 0 and g(1) = 2. If x [0, 1], 0 = g(0) < g(x) < g(1) = 2, so
g maps [0, 1] into [0, 2]. By the intermediate value theorem, every point in
[0, 2] is in the image of g. Thus, g is a bijection from [0, 1] to [0, 2].
The fact that h = g
1
is continuous is a general fact about strictly monotone
functions that follows from the intermediate value theorem. For complete-
ness, well give a proof here, but you should probably skip it on a rst
reading.
Lemma 0.2. Let f : I R be a continuous strictly increasing function,
where I R is an interval. Then the image of an interval J I is an
interval of the same type (i.e., open, closed, etc.) In particular, f(I) is an
interval of the same type as I.
Proof of Lemma. Let (a, b) I be an open interval. If x (a, b) then
a < x < b and f(a) < f(x) < f(b), since f is strictly increasing. Thus,
f((a, b)) (f(a), f(b)). On the other hand, if y (f(a), f(b)) then there is
an x (a, b) such that f(x) = y, by the intermediate value theorem. Thus,
f maps the open interval (a, b) onto the open interval (f(a), f(b)).
Suppose that [a, b) I is a half-open interval. By the previous part of the
proof (a, b) gets mapped onto (f(a), f(b)), and a certainly gets mapped onto
f(a). Thus, f maps [a, b) onto [f(a), f(b)). The other types of intervals are
dealt with in a similar fashion.
Proposition 0.3. Let f : I R be a continuous strictly increasing function,
where I R is an interval, and let J = f(I). Then the inverse function
f
1
: J I is continuous.
Proof. We need to show that if U I is open relative to I, then (f
1
)
1
(U) =
f(U) is open relative to J. Every open set in J can be written as a union of
bounded relatively open intervals, and the bijection f preserves unions, so it
will suce to show that the image of a bounded relatively open interval is
open in J.
If (a, b) I, then (a, b) is open relative to I, and f((a, b)) = (f(a), f(b)) (by
the lemma) which is open relative to J. If I has a lower endpoint, call it a,
then intervals of the form [a, b) I are open relative to I. By the lemma,
17
f(a) is the lower endpoint of J, and f([a, b)) = [f(a), f(b)), which is open
relative to J. The case where I has an upper endpoint is dealt with similarly.
There are similar results (with the obvious modications) for strictly de-
creasing functions.
b. If C is the Cantor set, m(g(C)) = 1.
Answer:
Recall that A = [0, 1] C is the union of a countably innite collection of
disjoint open intervals I
k

k=1
such that
m([0, 1] C) =

k=1
m(I
k
) = 1
(which is why C has measure zero). Also recall that the Cantor function is
dened to be constant on each of the closed intervals I
k
.
If I
k
= (a
k
, b
k
), then Cantor function f takes some constant value on
[a
k
, b
k
]. Then we have
g((a
k
, b
k
)) = (g(a
k
), g(b
k
))
= (a
k
+f(a
k
), b
k
+f(b
k
))
= (a
k
+, b
k
+),
which is an interval of the same length as I
k
. Thus,
m(g(A)) =

k=1
m(g(I
k
)) =

k=1
m(I
k
) = 1.
Since [0, 1] = A C (disjoint union), we have
[0, 2] = g([0, 1]) = g(A) g(C), (disjoint union)
and so
2 = m([0, 2]) = m(g(A)) +m(g(C)) = 1 +m(g(C)),
from which we conclude that m(g(C)) = 1.
c. By Exercise 29 of Chapter 1, g(C) contains a Lebesgue nonmeasurable set
A. Let B = g
1
(A). Then B is Lebesgue measurable but not Borel.
Answer:
Since A g(C), we have B = g
1
(A) C. Thus, B is a subset of the
Lebesgue nullset C, and so is Lebesgue measurable (since Lebesgue measure
is complete). On the other hand, if B was Borel, so would be g(B) =
(g
1
)
1
(B), since g
1
is continuous. But g(B) = A and A is not Lebesgue
measurable, let alone Borel. Thus, B is not Borel.
18
d. There exists a Lebesgue measurable function F and a continuous function G
on R so that F G is not Lebesgue measurable.
Answer:
Let the sets A [0, 2] and B C [0, 1] be as in the last part of the
problem.
Let F =
B
, which is Lebesgue measurable since B is Lebesgue measurable
and let G: [0, 2] [0, 1] be g
1
, which is continuous. The function F
G: [0, 2] R takes only the values 0 and 1 and
1 = (F G)(x) 1 = (
B
g
1
)(x)
1 =
B
(g
1
(x))
g
1
(x) B
x g(B) = A
1 =
A
(x)
Thus, F G =
A
, which is not Lebesgue measurable, since the set A is
nonmeasurable.
Actually, this example doesnt quite fulll the requirements of the problem,
since G is dened only on [0, 2], not all of R. To x this, extend the Cantor
function f from [0, 1] to R by dening f(x) = 0 for x < 0 and f(x) = 1
for x > 1. Then dene g on R by g(x) = f(x) + x. Now g is a strictly
increasing function R R, so g
1
: R R is continuous. On [0, 1] this g
agrees with our old g, so the above example will work with G = g
1
: R R
and F =
B
(dened on all of R).
19

You might also like